高考真题

\documentclass[11pt,space]{ctexart} % ans
\usepackage{GEEexam}
\usepackage{float}
\linespread{1.8}
%\watermark{60}{6}{14-金融工程-白兔兔} %水印

\everymath{\displaystyle}
\begin{document}\zihao{-4}
\juemi %输出绝密
\biaoti{2023年普通高等学校招生全国统一考试}
\fubiaoti{数$\quad$学}
本试卷共4页,22小题,满分150分。考试用时120分钟。\\
{\heiti 注意事项}:
\begin{enumerate}[itemsep=-0.3em,topsep=0pt]
\item 答卷前,考生务必将自己的姓名和准考证号填写在答题卡上。
\item 作答选择题时,选出每小题答案后,用2B铅笔把答题卡对应题目的答案标号涂黑。如需改动,用橡皮擦干净后,再选涂其它答案标号。在试卷上作答无效。
\item 非选择题必须使用黑色字迹的钢笔或签字笔作答,答案必须写在答题卡各题目指定区域内相应位置上。
\item 考生必须保持答题卡的整洁。考试结束后,将本试卷和答题卡一并交回。
\end{enumerate}
%%====================================================================
%%—————————————————————————————正文开始———————————————————————————————
%%====================================================================

\section{选择题:本大题共8小题,每小题5分,共40分。在每小题给出的四个选项中,只有一项是符合题目要求的。}

\begin{enumerate}[itemsep=0.3em,topsep=0pt]
\item 已知集合$M=\{-2,\ -1,\ 0,\ 1,\ 2\}$, $N=\{x\mid x^{2}-x-6\geqslant 0\}$,则$M\cap N=$
\begin{tasks}(4)
\task $\{-2,\ -1,\ 0,\ 1\}$ \task $\{0,\ 1,\ 2\}$
\task $\{-2\}$ \task $\{2\}$
\end{tasks}

\item 已知$z=\frac{1-\mathrm{i}}{2+2\mathrm{i}}$,则$z-\overline{z}=$
\begin{tasks}(4)
\task $-\mathrm{i}$ \task $\mathrm{i}$ \task $0$ \task $1$
\end{tasks}

\item 已知向量$\bm{a}=(1,1)$, $\bm{b}=(1,-1)$.若$(\bm{a}+\lambda \bm{b})\perp (\bm{a}+\mu \bm{b})$,则
\begin{tasks}(4)
\task $\lambda+\mu=1$ \task $\lambda+\mu=-1$ \task $\lambda\mu=1$ \task $\lambda\mu=-1$
\end{tasks}

\item 设函数$f(x)=2^{x(x-a)}$在区间$(0,1)$单调递减,则 $a$的取值范围是
\begin{tasks}(4)
\task $(-\infty,-2]$ \task $[-2,0)$ \task $(0,2]$ \task $[2,+\infty)$
\end{tasks}

\item 设椭圆$C_{1}:\frac{x^{2}}{a^{2}}+y^{2}=1\ (a>1)$, $C_{2}:\frac{x^{2}}{4}+y^{2}=1$的离心率分别为 $e_{1},e_{2}$.若$e_{2}=\sqrt{3}e_{1}$,则 $a=$
\begin{tasks}(4)
\task $\frac{2\sqrt{3}}{3}$ \task $\sqrt{2}$ \task $\sqrt{3}$ \task $\sqrt{6}$
\end{tasks}

\item 过点$(0,-2)$与圆$x^2+y^2-4x-1=0$相切的两条直线的夹角为$\alpha$,则$\sin\alpha=$
\begin{tasks}(4)
\task $1$ \task $\frac{\sqrt{15}} {4}$ \task $\frac{\sqrt{10}}{4}$ \task $\frac{\sqrt{6}}{4}$
\end{tasks}

\item 记$S_n$为数列$\{a_n\}$的前$n$项和,设甲: $\{a_n\}$为等差数列; 乙: $\left\{\frac{S_n}n\right\}$为等差数列,则
\begin{tasks}(2)
\task 甲是乙的充分条件但不是必要条件 \task 甲是乙的必要条件但不是充分条件 \task 甲是乙的充要条件 \task 甲既不是乙的充分条件也不是乙的必要条件
\end{tasks}

\item 已知$\sin(\alpha-\beta)=\frac{1}{3}$, $\cos\alpha\sin\beta=\frac{1}{6}$,则$\cos(2\alpha+2\beta)=$
\begin{tasks}(4)
\task $\frac{7}{9}$ \task $\frac{1}{9}$ \task $-\frac{1}{9}$ \task $-\frac{7}{9}$
\end{tasks}

\end{enumerate}

\section{选择题:本大题共4小题,每小题5分,共20分。在每小题给出的四个选项中,有多项是符合题目要求的。全部选对得5分,部分选对得2分,有选错的得0分。}

\begin{enumerate}[itemsep=0.3em,topsep=0pt]
\setcounter{enumi}{8}
\item 有一组样本数据$x_1,\ x_2,\ \cdots,\ x_6$,其中$x_1$是最小值, $x_{6}$ 是最大值,则
\begin{tasks}(1)
\task $x_2,\ x_3,\ x_4,\ x_5$ 的平均数等于$x_1,\ x_2,\ \cdots,\ x_6$ 的平均数
\task $x_2,\ x_3,\ x_4,\ x_5$ 的中位数等于$x_1,\ x_2,\ \cdots,\ x_6$ 的中位数
\task $x_2,\ x_3,\ x_4,\ x_5$的标准差不小于$x_1,\ x_2,\ \cdots,\ x_6$的标准差
\task $x_2,\ x_3,\ x_4,\ x_5$的极差不大于$x_1,\ x_2,\ \cdots ,\ x_6$的极差
\end{tasks}

\item 噪声污染问题越来越受到重视.用声压级来度量声音的强弱,定义声压级$L_p=20\times \lg\frac{p}{p_{0}}$,其中常数$p_{0}\ (p_{0}>0)$是听觉下限阈值, $p$是实际声压.下表为不同声源的声压级:
\begin{table}[H]
\centering
\setlength{\tabcolsep}{6mm}{
\begin{tabular}{|c|c|c|}
\hline
声源 & 与声源的距离$/$ m & 声压级$/$ dB \\ \hline
燃油汽车 & 10 & 60 $\sim$ 90 \\ \hline
混合动力汽车 & 10 & 50 $\sim$ 60 \\ \hline
电动汽车 & 10 & 40 \\ \hline
\end{tabular}}
\end{table}
己知在距离燃油汽车、混合动力汽车、电动汽车 10 m 处测得实际声压分别为$p_1,p_2,p_3$,则
\begin{tasks}(4)
\task $p_{1}\geqslant p_{2}$ \task $p_2> 10p_{3}$
\task $p_{3}= 100p_{0}$ \task $p_{1}\leqslant 100p_{2}$
\end{tasks}

\item 已知函数$f(x)$的定义域为$\mathbf{R}$, $f(xy)=y^2f(x)+x^2f(y)$,则
\begin{tasks}(2)
\task $f(0)=0$
\task $f(1)=0$
\task $f(x)$是偶函数
\task $x=0$为$f(x)$的极小值点
\end{tasks}

\item 下列物体中,能够被整体放入棱长为 1 (单位: m)的正方体容器(容器壁厚度忽略不计)内的有
\begin{tasks}(2)
\task 直径为$0.99$ m 的球体
\task 所有棱长均为$1.4$ m 的四面体
\task 底面直径为$0.01$ m, 高为$1.8$ m 的圆柱体
\task 底面直径为$1.2$ m, 高为$0.01$ m 的圆柱体
\end{tasks}

\end{enumerate}

\section{填空题:本题共4小题,每小题5分,共20分。}
\begin{enumerate}[itemsep=0.3em,topsep=0pt,resume]%\setcounter{enumi}{12}

\item 某学校开设了4门体育类选修课和4门艺术类选修课,学生需从这 8门课中选修 2门或 3 门课,并且每类选修课至少选修 1 门,则不同的选课方案共有\blank{}种(用数字作答).
\item 在正四棱台$ABCD-A_1B_1C_1D_1$中, $AB=2,A_1B_1=1,AA_1=\sqrt{2}$,则该棱台的体积为\blank{}.
\item 已知函数$f(x)=\cos\omega x-1\ (\omega>0)$在区间$[0,2\pi]$有且仅有$3$个零点,则$\omega$的取值范围是\blank{}.
\item 已知双曲线$C:\frac{x^2}{a^2}-\frac{y^2}{b^2}=1\ (a>0,b>0)$的左、右焦点分别为$F_1,F_2$.点$A$在$C$上,点$B$在$y$轴上, $\overrightarrow{F_1A}\perp\overrightarrow{F_1B}$, $\overrightarrow{F_2A}=-\frac23\overrightarrow{F_2B}$,
则$C$的离心率为\blank{}.

\end{enumerate}

\section{解答题:本题共6小题,共70分。解答应写出文字说明、证明过程或演算步骤。}

\begin{enumerate}[itemsep=0.5em,topsep=5pt,resume]%\setcounter{enumi}{17}
\item (10分)\\
已知在$\triangle ABC$中, $A+B=3C$, $2\sin(A-C)=\sin B$. \begin{enumerate}[itemsep=-0.3em,label={(\arabic*)},topsep=0pt,labelsep=.5em,leftmargin=3em]
\item 求$\sin A$;
\item 设$AB=5$,求$AB$边上的高.
\end{enumerate}


\begin{minipage}[h]{.3\textwidth}
\item (12分)\\
如图,在正四棱柱 $ABCD-A_1B_1C_1D_1$中, $AB=2$, $AA_1=4$.点$A_2,B_2,C_2,D_2$分别在棱$AA_1$, $BB_1$, $CC_{1}$, $DD_{1}$上, $AA_{2}=1$, $BB_{2}=DD_{2}=2$, $CC_{2}=3$.
\begin{enumerate}[itemsep=-0.3em,label={(\arabic*)},topsep=0pt,labelsep=.5em,leftmargin=3em]
\item 证明: $B_2C_2// \ A_2D_2$;
\item 点$P$在棱$BB_1$上,当二面角$P-A_{2}C_{2}-D_{2}$为$150^\circ$时,求$B_{2}P$.
\end{enumerate}
\end{minipage}\hspace{0.5em}
\begin{minipage}[h]{.2\textwidth}
\begin{figure}[H]
\includegraphics[width=6cm]{19titu.png}
\end{figure}
\end{minipage}\vspace{1em}


\item (12分)\\
已知函数$f(x)=a\left(\mathrm{e}^{x}+a\right)-x$.
\begin{enumerate}[itemsep=-0.3em,label={(\arabic*)},topsep=0pt,labelsep=.5em,leftmargin=3em]
\item 讨论$f(x)$的单调性;
\item 证明:当$a>0$时, $f(x)>2\ln a+\frac{3}{2}$.
\end{enumerate}

\item (12分)\\
设等差数列$\{a_n\}$的公差为$d$,且$d>1$.令$b_n=\frac{n^2+n}{a_n}$,记$S_n,T_n$分别为数列$\{a_n\}$, $\{b_n\}$的前$n$项和.
\begin{enumerate}[itemsep=-0.3em,label={(\arabic*)},topsep=0pt,labelsep=.5em,leftmargin=3em]
\item 若$3a_{2}=3a_{1}+a_{3}$, $S_{3}+T_{3}=21$, 求$\{a_n\}$的通项公式;
\item 若$\{b_n\}$为等差数列,且$S_{99}-T_{99}=99$,求$d$.
\end{enumerate}

\begin{solution}
(1)由题设$3a_{2}=3a_{1}+a_{3}$,可知 $3a_{1}+3d=3a_{1}+a_{1}+2d$,解得首项与公差的关系 $a_{1}=d$.

再由题设$S_{3}+T_{3}=21$得$6d+\frac{9}{d}=21$,解得 $d=\frac{1}{2}$ (舍去)或$d=3$.

因此, $\{a_n\}$的通项公式为$a_n=3n$.

(2) \textbf{解法1:}由题设,数列$\{a_n\}$, $\left\{\frac{n^2+n}{a_n}\right\}$均为等差数列,故$\frac{6}{a_1+d}=\frac{1}{a_1}+\frac{6}{a_{1}+2d}$,
解得$a_{1}=d$或$a_{1}=2d$.

(i)当$a_{1}=2d$时, $S_{n}=\frac{n(n+3)d}{2}$, $T_{n}=\frac{n(n+1)}{2d}$.
又$d>1$,故$S_{n}-T_{n}>n$,与题设不符.

(ii)当$a_1=d$时, $S_n=\frac{n(n+1)d}{2}$, $T_n=\frac{n(n+3)}{2d}$, 从而$\frac{99\times 100 d}{2}-\frac{99\times102}{2d}=99$,解得$d=-1$ (舍去)或$d=\frac{51}{50}$.因此 $d=\frac{51}{50}$.

\textbf{解法2:} 由题设可得$b_{n+1}-b_{n}=\frac{(n+1)^{2}+n+1}{a_{n+1}}
-\frac{n^{2}+n}{a_{n}}$.

又$\{b_n\}$为等差数列,令$\frac{(n+1)^2+n+1}{a_{n+1}}-\frac{n^2+n}{a_{n}}=m$,则对任意正自然数$n$均有
$$
n^{2}\left(md^{2}-d\right)+\left(2a_{1}md-md^{2}
-2a_{1}+d\right)n+\left(ma_{1}^{2}-mda_{1}
-2a_{1}+2d\right)=0.
$$
故$md^{2}-d=0$, $2a_{1}md-md^{2}-2a_{1}+d=0$,
且$ma_{1}^{2}-mda_{1}-2a_{1}+2d=0$.

由题设及$md^{2}-d=0$,解得$m=\frac{1}{d}$,故 $ma_{1}^{2}-mda_{1}-2a_{1}+2d=
\frac1d({a_1}^2-3a_1d+2d^2)=0$.

由$a_{1}^{2}-3a_{1}d+2d^{2}=0$,
解得$a_{1}=2d$或$a_{1}=d$.

余下部分同解法 1.
\end{solution}

 

\item (12分)\\
甲、乙两人投篮,每次由其中一人投篮,规则如下:若命中则此人继续投篮,若未命中则换为对方投篮. 无论之前投篮情况如何,甲每次投篮的命中率均为$0.6$, 乙每次投篮的命中率均为$0.8$. 由抽签确定第 $1$次投篮的人选,第 1 次投篮的人是甲、乙的概率各为$0.5$.
\begin{enumerate}[itemsep=-0.3em,label={(\arabic*)},topsep=0pt,labelsep=.5em,leftmargin=3em]
\item 求第2次投篮的人是乙的概率;
\item 求第$i$次投篮的人是甲的概率;
\item 已知:若随机变量$X_i$服从两点分布,且$P(X_i=1)=1-P(X_i=0)=q_i$, $i=1,2,\cdots,n$,则$E\left(\sum_{i=1}^nX_i\right)=\sum_{i=1}^nq_i$.记前$n$次(即从第 1 次到第 $n$次投篮)中甲投篮的次数为$Y$,求 $E(Y)$.
\end{enumerate}
\begin{solution}
(1)第2次投篮的人是乙,共有下面两种情况:第1次投篮的人是甲且甲未命中,概率为$0.5\times (1-0.6)=0.2$;

第1次投篮的人是乙且乙命中,概率为$0.5\times 0.8=0.4$.

所以第2次投篮的人是乙的概率为$0.2+0.4=0.6$.

(2)设$A_k$表示事件“第$k$次投篮的人是甲”, 记$p_k=P(A_k)$, $k=1,2,\cdots,i$.

当$k\geqslant 2$时,第$k$次投篮的人是甲,共有下面两种情况:

第 $k-1$次投篮的人是甲且甲命中,概率为 $P(A_{k-1})\times 0.6=\frac{3}{5}p_{k-1}$;

第$k-1$次投篮的人是乙且乙未命中,概率为$P\left(\overline{A_{k-1}}\right)\times(1-0.8)=\frac{1}{5}(1-p_{k-1})$.

所以当$k\geqslant 2$时, $p_ {k}=\frac{3}{5}p_{k-1}+\frac{1}{5}(1-p_{k-1})
=\frac{2}{5}p_{k-1}+\frac{1}{5}$,即
$$
p_k-\frac{1}{3}
=\frac{2}{5}\left(p_{k-1}-\frac{1}{3}\right),\quad
\text{得} p_k=\frac{1}{3}+\left(p_1-\frac{1}{3}\right)
\times\left(\frac{2}{5}\right)^{k-1}.
$$

又$p_{1}=\frac{1}{2}$,所以第$i$次投篮的人是甲的概率为$p_{i}=\frac{1}{3}+\frac{1}{6}
\times\left(\frac{2}{5}\right)^{i-1}$.

(3)设随机变量$X_i=\begin{cases}1, &\text{第$i$次投篮的人是甲,}\\
0,&\text{第$i$次投篮的人是乙}\end{cases}(i=1,2,\cdots,n)$,则
$X_{i}$服从两点分布, $P(X_ {i}=1)=\frac{1}{3}+\frac{1}{6}\times
\left(\frac{2}{5}\right)^{i-1}$,且 $Y=\sum_{i=1}^{n}X_{i}$.由题设可得
$$E(Y)=E\left(\sum_ {i=1}^{n}X_{i}\right)
=\sum_{i=1}^{n}\left[\frac{1}{3}
+\frac{1}{6}\times
\left(\frac{2}{5}\right)^{i-1}\right]
=\frac{n}{3}+\frac{5}{18}
\left[1-\left(\frac{2}{5}\right)^{n}\right].$$
\end{solution}


\item (12分) \\
在直角坐标系$xOy$中,点$P$到$x$轴的距离等于点$P$到点$\left(0,\frac12\right)$的距离,记动点$P$的轨迹为$W$.
\begin{enumerate}[itemsep=-0.3em,label={(\arabic*)},topsep=0pt,labelsep=.5em,leftmargin=3em]
\item 求$W$的方程;
\item 已知矩形$ABCD$有三个顶点在$W$上,证明:矩形$ABCD$的周长大于$3\sqrt{3}$.
\end{enumerate}
\begin{solution}
1
\end{solution}

 

\item (12分) \\
已知双曲线$C$的中心为坐标原点,左焦点为$(-2\sqrt{5},0)$,离心率为$\sqrt{5}$.
\begin{enumerate}[itemsep=-0.3em,label={(\arabic*)},topsep=0pt,labelsep=.5em,leftmargin=3em]
\item 求$C$的方程;
\item 记$C$的左、右顶点分别为$A_1,A_2$,过点$(-4,0)$的直线与$C$的左支交于$M$, $N$两点, $M$在第二象限,直线$MA_{1}$与$NA_{2}$交于点$P$, 证明: 点$P$在定直线上.
\end{enumerate}
\begin{solution}
1
\end{solution}


\item (12分) %\\
\begin{enumerate}[itemsep=-0.3em,label={(\arabic*)},topsep=0pt,labelsep=.5em,leftmargin=3em]
\item 证明:当$0<x<1$时, $x-x^2<\sin x<x$;
\item 已知函数$f(x)=\cos ax-\ln(1-x^2)$,若$x=0$是$f(x)$的极大值点,求$a$的取值范围.
\end{enumerate}
\begin{solution}
(1)令$g(x)=x-\sin x$. $g'(x)=1-\cos x\geqslant 0$,故$g(x)$单调递增.当$x>0$时, $g(x)>g(0)=0$, 即 $\sin x<x$.

令$F(x)=x-x^{2}-\sin x$.当$0<x<1$时,
$$
F'(x)=1-\cos x-2x=2\sin^{2}\frac{x}{2}
-2x<2\left(\frac{x}{2}\right)^{2}-2x<0,
$$
故$F(x)$单调递减, $F(x)<F(0)=0$,即$x-x^2<\sin x$.

综上,当$0<x<1$时, $x-x^2<\sin x<x$.
\end{solution}


\begin{solution}
(2)若$x=0$是$f(x)$的极大值点,则存在$\delta\in(0,1)$,使得当$x\in(-\delta,0)\cup (0,\delta)$时, $f(x)<f(0)=0$,为了找到满足题意的$\delta$,要通过$f(x)$的导函数的符号讨论$f(x)$的单调性.

因为$f(x)$是偶函数,不妨设$0<x<1$,又因为$\cos ax=\cos(-ax)$,不妨设$a\geqslant 0$.

$$
f'(x)=\frac{2x-a(1-x^2)\sin ax}{1-x^2},\quad x\in (-1,1),
$$

分母$1-x^2>0$,只需讨论分子$2x-a(1-x^2)\sin ax$的符号.令 $h(x)=2x-a\left(1-x^{2}\right)\sin ax$.

第(1)问中的不等式为第(2)问做好了准备工作.

当$x>0$时, $\sin x<x$, 于是$h(x)\geqslant 2x-a^2x(1-x^2)=x(2-a^2+a^2x^2)$.

当$0<x<1$时, $x-x^2<\sin x$,于是 $h(x)<2x-a^2x(1-x^2)(1-ax)$.

如果找到$\delta\in(0,1)$,使得当$x\in(0,\delta)$时, $h(x)>0$,则$f'(x)>0$, 故$f(x)$在$(0,\delta)$单调递增,从而$f(x)>f(0)$,那么$x=0$就不是$f(x)$的极大值点.要使$h(x)>0$,只需$x(2-a^2+a^2x^2)>0$,
又$a^2x^2>0$,所以只需考虑$2-a^{2}$的符号,于是找到了分类讨论的标准$\sqrt{2}$.

当$0\leqslant a\leqslant\sqrt{2}$时, $h\left(x\right)\geqslant x\left(2-a^{2}+a^{2}x^{2}\right)>0$.当$0<x<1$时, $h(x)>0$, $f'(x)>0$,故$f(x)$单调递增,从而$f(x)>f(0)$,因此$x=0$不是$f(x)$的极大值点.

当$a>\sqrt{2}$时, $h(x)<2x-a^2x(1-x^2)(1-ax)$,为了找到$\delta\in(0,1)$,使得当$x\in(0,\delta)$时, $h(x)<0$,只需$2x-a^2x(1-x^2)(1-ax)<0$,即$(1-x^2)\cdot (1-ax)>\frac{2}{a^{2}}$.此不等式不容易解,继续进行不等式放缩:
$$
(1-x^2)(1-ax)>(1-x)(1-ax)>(1-ax)^2,
$$
只需$(1-ax)^{2}>\frac{2}{a^{2}}$,解得 $1-ax>\frac{\sqrt{2}}{a}$,即 $0<x<\frac{a-\sqrt{2}}{a^{2}}$.
于是当$0<x<\frac{a-\sqrt{2}}{a^2}$时,
$$
h(x)<2x-a^2x(1-x^2)(1-ax)<x[2-a^2(1-ax)^2]<0,
$$
从而$f'(x)<0$,故$f(x)$在$\left(0,\frac{a-\sqrt{2}}{a^2}\right)$单调递减.

又因为$f(x)$是偶函数,故$x=0$是$f(x)$的极大值点.

综上, $a$的取值范围是$\left(-\infty,-\sqrt{2}\right) \cup\left(\sqrt{2},+\infty\right)$.
\end{solution}


\item (12分) \\
已知椭圆$C:\frac{x^2}{a^2}+\frac{y^2}{b^2}=1\ (a>b>0)$的离心率为$\frac{\sqrt{5}}{3}$,点$A(-2,0)$在$C$上.
\begin{enumerate}[itemsep=-0.3em,label={(\arabic*)},topsep=0pt,labelsep=.5em,leftmargin=3em]
\item 求$C$的方程;
\item 过点$(-2,3)$的直线交$C$于$P$, $Q$两点,直线 $AP$, $AQ$与$y$轴的交点分别为$M$, $N$,证明:线段$MN$的中点为定点.
\end{enumerate}
\begin{solution}
1
\end{solution}

 

\item (12分) \\
已知函数$f(x)=\left(\frac{1}{x}+a\right)\ln(1+x)$.
\begin{enumerate}[itemsep=-0.3em,label={(\arabic*)},topsep=0pt,labelsep=.5em,leftmargin=3em]
\item 当$a=-1$时,求曲线 $y=f(x)$在点$(1,f(1))$处的切线方程;
\item 是否存在$a$, $b$,使得曲线 $y=f\left(\frac1x\right)$关于直线$x=b$对称?若存在,求$a,b$;若不存在,说明理由;
\item 若$f(x)$在$(0,+\infty)$存在极值点,求$a$的取值范围.
\end{enumerate}
\begin{solution}
(1)当$a=-1$时, $f(x)=\left(\frac1x-1\right)\ln( 1+x)$, $f'( x) =-\frac1{x^{2}}\ln (1+x)+\left(\frac{1}{x}-1\right)\frac{1}{1+x}$,所以$f(1)=0$, $f'(1)=-\ln2$.

曲线$y=f(x)$在点$(1,f(1))$处的切线方程为 $y=-\ln2\cdot(x-1)$.

(2) \textbf{解法1:} 设$g(x)=f\left(\frac1x\right)$,则$g(x)=(x+a)\ln\left(1+\frac1x\right)$, $g(x)$的定义域为$(-\infty,-1)\cup (0,+\infty)$.

若存在$a$, $b$,使得曲线$y=g(x)$关于直线$x=b$对称,则$(-\infty,-1)\cup (0,+\infty)$关于$x=b$对称,所以$b=-\frac{1}{2}$.

由$g(x)=g(-1-x)$得$(x+a)\ln\left(1+\frac{1}{x}\right)=(-1-x+a)\ln\frac{x}{1+x}$,整理得
$$
(x+a)\ln\left(1+\frac{1}{x}\right)
=(x+1-a)\ln\left(1+\frac{1}{x}\right),
$$

所以 $a=\frac{1}{2}$, 故存在 $a=\frac{1}{2},b=-\frac{1}{2}$,使得曲线 $y=f\left(\frac{1}{x}\right)$关于直线$x=b$对称.

\textbf{解法2:} 设函数$g\left(x\right)=f\left(\frac{1}{x}\right)$,则 $g\left(x\right)=\left(x+a\right)
\ln\left(1+\frac{1}{x}\right)$.若曲线
$y=f\left(\frac{1}{x}\right)$关于直线$x=b$对称,则 $g(x)=g(2b-x)$,即
$$(x+a)\ln\left(1+\frac{1}{x}\right)
=(2b-x+a)\ln\left(1+\frac{1}{2b-x}\right).$$

将上式恒等变形为
$$(x+a)\ln{\frac{x+1}{x}}
=(x-2b-a)\ln{\frac{x-2b}{x-2b-1}},$$
由此可以得到,当
$\begin{cases}
-2b=1,\\
-2b-1=0,\\
-2b-a=a,
\end{cases}$
即$a=\frac{1}{2},b=-\frac{1}{2}$时,曲线 $y=f\left(\frac{1}{x}\right)$关于直线$x=b$对称.
\end{solution}


\begin{solution}
(3) $f'(x)=-\frac{1}{x^{2}}\ln(1+x)
+\left(\frac{1}{x}+a\right)\cdot\frac{1}{1+x}
=-\frac{1}{x^{2}}\left(\ln(1+x)
-\frac{ax^{2}+x}{1+x}\right)$.

令$h(x)=\ln(1+x)-\frac{ax^2+x}{1+ x}$,则$h(0)=0$,且
$$
h'(x)=\frac{1}{1+x}-\frac{(2ax+1)(1+x)-(ax^{2}+x)}
{(1+x)^{2}}=\frac{x(1-2a-ax)}{(1+x)^{2}}.
$$

(i)若$a\leqslant0$,当$x\in(0,+\infty)$时, $h'(x)>0$, $h(x)$在$(0,+\infty)$单调递增,从而$h(x)>h(0)=0$.故当$x\in(0,+\infty)$时, $f'(x)<0$, $f(x)$在$(0,+\infty)$单调递减, $f(x)$在$(0,+\infty)$不存在极值点.

(ii)若$a\geqslant\frac{1}{2}$,当$x\in(0,+\infty)$时, $h'(x)<0$, $h(x)$在$(0,+\infty)$单调递减,从而$h(x)<h(0)=0$.故当$x\in(0,+\infty)$时, $f'(x)>0$, $f(x)$在$(0,+\infty)$单调递增, $f(x)$在$(0,+\infty)$不存在极值点.

(iii)若$0<a<\frac{1}{2}$,当 $x\in\left(0,\frac{1}{a}-2\right)$时, $h'(x)>0$, $h(x)$在$\left(0,\frac{1}{a}-2\right)$单调递增;
当$x\in\left({\frac{1}{a}}-2,+\infty\right)$时, $h'(x)<0$, $h(x)$在$\left({\frac{1}{a}}-2,+\infty\right)$单调递减.所以当$x\in\left(0,\frac{1}{a}-2\right]$时, $h(x)>h(0)=0$.


取$x_1=\mathrm{e}^t-1$,其中常数$t>\frac4a$,则$x_1\in\left(\frac1a-2,+\infty\right)$,且
\begin{align*}
h\left( x_1 \right) &=t-\frac{a\left( \text{e}^t-1 \right) ^2+\text{e}^t-1}{\text{e}^t}<t+1-a\text{e}^t<t+1
-a\left(1+\frac{t}{2} \right) ^2\\
&=\left( t+2 \right) \left[ \frac{t+1}{t+2}-\frac{a}{4}\left( t+2 \right) \right] <0,
\end{align*}
所以$h\left(x\right)$在$\left(\frac1a-2,+\infty\right)$
有唯一零点$x_0$.当$x\in(0,x_0)$时, $h(x)>0$, $f'(x)<0$, $f(x)$在$(0,x_0)$单调递减;当$x\in(x_0,+\infty)$时, $h(x)<0$, $f'(x)>0$, $f(x)$在$(x_0,+\infty)$单调递增,因此$f(x)$在$(0, +\infty)$存在极值点 $x_{0}$.

综上, $a$的取值范围是$\left(0,\frac12\right)$.
\end{solution}

 

\item (12分) \\
已知函数$f(x)=ax-\frac{\sin x}{\cos^{3}x}$, $x\in\left(0,\frac{\pi}{2}\right)$.
\begin{enumerate}[itemsep=-0.3em,label={(\arabic*)},topsep=0pt,labelsep=.5em,leftmargin=3em]
\item 当$a=8$时,讨论$f(x)$的单调性;
\item 若$f(x)<\sin 2x$,求$a$的取值范围.
\end{enumerate}
\begin{solution}
$f'\left(x\right)=a-\frac{\cos^{4}x+3\sin^{2}x\cos^{2}x}
{\cos^{6}x}=a-\frac{3-2\cos^{2}x}{\cos^{4}x}$.

(1)当$a=8$时, $f'(x)=8-\frac{3-2\cos^{2}x}{\cos^{4}x}
=\frac{(2\cos^{2}x-1)(4\cos^{2}x+3)}{\cos^{4}x}$,
当$x\in\left(0,\frac{\pi}{4}\right)$时, $f'(x)>0$, $f(x)$在区间$\left(0,\frac{\pi}{4}\right)$单调递增;

当$x\in\left(\frac{\pi}{4},\frac{\pi}{2}\right)$时, $f'(x)<0$, $f(x)$在区间$\left(\frac\pi4,\frac\pi2\right)$单调递减.

(2) \textbf{解法1:} 若$a\leqslant 0$,则$f(x)<\sin 2x$.下设$a>0$.

当$x\in\left(0,\frac{\pi}{2}\right)$时, $f(x)<\sin 2x$,当且仅当$\frac1{\cos^3x}+2\cos x>a\frac{x}{\sin x}$.

因为当$x\in\left(0,\frac\pi2\right)$时, $\sin x<x$,所以当$\frac1{\cos^3x}+2\cos x>a\frac x{\sin x}$时,有
$$
\frac1{\cos^3x}+2\cos x>a.
$$

设$g(t)=\frac{1}{t^3}+2t$,则$g'(t)=2-\frac{3}{t^4}$.
当$t\in(0,1)$时, $g'(t)<0$,故$g(t)$在$(0,1)$单调递减.所以当$t\in(0,1)$时, $g(t)>g(1)=3$. 故$x\in \left(0,\frac\pi2\right)$时,函数$\frac1{\cos^3x}+2\cos x$的取值范围为$(3, +\infty)$,所以 $a\leqslant 3$.

当$a\leqslant 3$时, $f(x)-\sin2x \leqslant 3x-{\frac{\sin x}{\cos^{3}x}}-\sin2x$.
设$h(x)=3x-{\frac{\sin x}{\cos^{3}x}}-\sin2x$,则
$h'(x)=3-\frac{3-2\cos^{2}x}{\cos^{4}x}-2\cos2x
=\frac{-4\cos^{6}x+5\cos^{4}x
+2\cos^{2}x-3}{\cos^{4}x}$.

令$F(t)=-4t^3+5t^2+2t-3$,则$F'(t)=-12t^2+10t+2=2(1-t)(6t+1)$.

当$t\in(0,1)$时, $F'(t)>0$,故$F(t)$在$(0,1)$单调递增.所以当$t\in(0,1)$时, $F(t)<F(1)=0$.

故当$x\in\left(0,\frac\pi2\right)$时, $-4\cos ^6x+5\cos ^4x+2\cos^2x-3<0$,从而$h'( x)<0$,所以$h(x)$在$\left(0,\frac\pi2\right)$单调递减.故当$x\in\left(0,\frac\pi2\right)$时, $h(x)<h(0)=0$,所以$f(x)<\sin 2x$.

综上, $a$的取值范围是$(-\infty,3]$.
\end{solution}


\begin{solution}
\textbf{解法2:}令$g(x)=3x-2\sin x-\tan x, x\in \left ( 0, \frac \pi2\right )$,
则$g'(x)=\frac{3\cos^{2}x-2\cos^{3}x-1}{\cos ^{2}x}$.

令$h(x)=-2\cos^{3}x+3\cos^{2}x-1, 0<x<\frac{\pi}{2}$,则$h'(x)=6\sin x\cos x(\cos x-1)<0$.

$h(x)$在区间$\left(0,\frac\pi2\right)$单调递减,所以$h(x)<h(0)=0$.从而$g'(x)<0$, $g(x)$在区间$\left(0,\frac{\pi}{2}\right)$单调递减,故$g(x)<g(0)=0$.

当$a\leqslant 3$ 时,
\begin{align*}
f(x)-\sin2x &\leqslant 3x-\frac{\sin x}{\cos^{3}x}-\sin2x\\
&<2\sin x+\tan x-\frac{\sin x}{\cos^{3}x}-2\sin x\cos x\\
&=\sin x(1-\cos x)\left(2-\frac{1}{\cos^{2}x} -\frac{1}{\cos^{3}x}\right)<0.
\end{align*}

当$a>3$时, 取$x_0\in\left(0,\frac\pi2\right)$, 满足 $\cos x_0>\frac1{\sqrt[3]{a-2}}$,又因为当$x\in \left(0,\frac{\pi}{2}\right)$时, $\sin x<x$,所以
\begin{align*}
f(x_{0})-\sin 2x_{0} &=ax_{0}-\frac{\sin x_{0}}{\cos^{3}x_{0}}-2\sin x_{0}\cos x_{0}\\
&\geqslant\sin x_{0}\left(a-\frac{1}{\cos^{3}x_{0}}-2\cos x_{0}\right) \\
&\geqslant\sin x_{0}\left(a-2-\frac{1}{\cos^{3}x_{0}}\right) >0.
\end{align*}

综上, $a$的取值范围是$(-\infty,3]$.
\end{solution}

 


\item (12分) \\
(2024年石家庄二模)设集合$M$是一个非空数集,对任意$x,y\in M$,定义$\rho(x,y)=|x-y|$,称$\rho$为集合$M$的一个度量,称集合$M$为一个对于度量$\rho$而言的度量空间,该度量空间记为$(M,\rho)$.

定义1:若 $f:M\to M$是度量空间$(M,\rho)$上的一个函数,且存在 $\alpha\in(0,1)$,使得对任意$x,y\in M$,均有: $\rho(f(x),f(y))\leqslant\alpha\rho(x,y)$,则称$f$是度量空间$(M,\rho)$上的一个“压缩函数”.

定义2:记无穷数列$a_0,a_1,a_2,\cdots$为$\left\{a_n\right\}_{n=0}^{+\infty}$,若$\left\{a_n\right\}_{n=0}^{+\infty}$是度量空间$\left(M,\rho\right)$上的数列,且对任意正实数 $\varepsilon>0$,都存在一个正整数$N$,使得对任意正整数 $m,n\geqslant N$,均有 $\rho(a_m,a_n)<\varepsilon$,则称$\{a_n\}_{n=0}^{+\infty}$是度量空间$(M,\rho)$上的一个“基本数列”.

\begin{enumerate}[itemsep=-0.3em,label={(\arabic*)},topsep=0pt,labelsep=.5em,leftmargin=3em]
\item 设 $f(x)=\sin x+\frac12$,证明: $f$是度量空间$\left(\left[\frac12,2\right],\rho\right)$上的一个“压缩函数”;
\item 已知 $f:\mathbf{R}\to\mathbf{R}$是度量空间 $(\mathbf{R},\rho)$上的一个压缩函数,且 $a_0\in\mathbf{R}$,定义 $a_{n+1}=f\left(a_n\right)$, $n=0,1,2,\cdots$,证明: $\left\{a_{n}\right\}_{n=0}^{+\infty}$为度量空间$\left(\mathbf{R},\rho\right)$上的一个“基本数列”.
\end{enumerate}
\begin{solution}
1
\end{solution}

 


\item (12分) \\
(2021年北京高考)设$p$为实数.若无穷数列$\{a_n\}$满足如下三个性质,则称$\{a_n\}$为$\Re_p$数列:

\ding{172} $a_1+p\geqslant 0$,且$a_2+p=0$;

\ding{173} $a_{4n-1}<a_{4n}\quad(n=1,2,\cdots)$;

\ding{174} $a_{m+n}\in\{a_m+a_n+p, a_m+a_n+p+1\}\quad (m=1,2,\cdots; n=1,2,\cdots)$.
\begin{enumerate}[itemsep=-0.3em,label={(\arabic*)},topsep=0pt,labelsep=.5em,leftmargin=3em]
\item[(I)] 如果数列$\{a_n\}$的前四项为$2,-2,-2,-1$,那么$\{a_n\}$是否可能为$\Re_{2}$数列?说明理由;
\item[(II)] 若数列$\{a_n\}$是$\Re_{0}$数列,求$a_5$;
\item[(III)] 设数列$\{a_{n}\}$的前$n$项和为$S_n$.是否存在$\Re_p$数列$\{a_{n}\}$,使得$S_n\geqslant S_{10}$恒成立?如果存在,求出所有的$p$;如果不存在,说明理由.
\end{enumerate}
\begin{solution}
(I)数列$\{a_n\}$不可能为$\Re_{2}$数列.理由如下:

因为$p=2$, $a_1=2$, $a_2=-2$, 所以$a_1+a_2+p=2$, $a_1+a_2+p+1=3$.因为$a_{3}=-2$,所以$a_{3}\notin\{a_{1}+a_{2}+p,a_{1}+a_{2}+p+1\}$.

所以数列$\{a_n\}$不满足性质\ding{174}.

(II)根据$\Re_{0}$数列的定义,可知$\{a_n\}$满足:
$a_1\geqslant0,a_2=0$; $a_{4n-1}<a_{4n}$; $a_{m+n}=a_n+a_n$或$a_{m+n}=a_m+a_n+1$.

由$a_{n+1}=a_n+a_1$或$a_{n+1}=a_n+a_1+1$,以及$a_1\geqslant 0$,可知$a_{n+1}\geqslant a_n$.
所以$a_1=0$.

由$a_3=a_1+a_2=0$或$a_3=a_1+a_2+1=1$;\quad $a_4=a_2+a_2=0$或$a_4=a_2+a_2+1=1$,以及$a_3<a_4$,可知$a_3=0,a_4=1$.

由$a_{5}=a_{2}+a_{3}=0$或$a_{5}=a_{2}+a_{3}+1=1$, 以及$a_5\geqslant a_4$,可知$a_5=1$.
\end{solution}


\begin{solution}
(III)假设数列 $\{a_n\}$ 是满足“$S_n\geqslant S_{10}$恒成立”的$\Re_{p}$数列.

因为$a_{n+1}=a_n+a_1+p$ 或$a_{n+1}=a_n+a_1+p+1$, 且$a_1+p\geqslant0$,所以$a_{n+1}\geqslant a_n$.

由$-p\leqslant a_{1}\leqslant a_{2}=-p$,可知$a_{1}=-p$.

从而$a_{4n}=a_{4n-1}+a_1+p=a_{4n-1}$
或$a_{4n}=a_{4n-1}+a_1+p+1=a_{4n-1}+1$.

又因为$a_{4n-1}<a_{4n}$,所以$a_{4n}=a_{4n-1}+1$.

因为$a_4=a_3+1$,且$a_3\geqslant a_2=-p$,所以$a_4\geqslant -p+1$.

又因为$a_4\leqslant a_2+a_2+p+1=-p+1$,所以$a_{4}=-p+1,a_{3}=-p$.

因为$a_{12}\leqslant a_{6}+a_{6}+p+1$,且$a_6\leqslant a_3+a_3+p+1=-p+1$,所以$a_{12}\leqslant-p+3$.

因为$a_{11}=a_{12}-1$,所以$a_{11}\leqslant -p+2$.

由$S_{11}\geqslant S_{10}$可知$a_{11}\geqslant 0$,所以$p\leqslant2$.

由$a_{10}\geqslant a_8=a_7+1$及$a_7\geqslant a_4=-p+1$, 可知$a_{10}\geqslant -p+2$.

由$S_{9}\geqslant S_{10}$可知$a_{10}\leqslant 0$,所以$p\geqslant 2$.

综上可知,若数列$\{a_n\}$是满足“$S_n\geqslant S_{10}$恒成立”的$\Re_p$数列,则$p=2$.


当$p=2$时,考虑数列$\{a_n\}$:
$$
a_n=\begin{cases}
-2+k,&n\in\{4k+1,4k+2,4k+3\},\\
-1+k,&n=4k+4
\end{cases}(k\in\mathbf{N}).
$$
下面验证数列$\{a_n\}$满足性质\ding{172} \ding{173} \ding{174}.

由$a_1=-2,a_2=-2$可知$a_1+p\geqslant 0,a_2+p=0$.

因为$a_{4n-1}=n-3,a_{4n}=n-2$,所以$a_{4n-1}<a_{4n}$.

对于任意正整数$m,n$,存在$k_1,k_2\in\mathbb{N},r_1,r_2\in\{0,1,2,3\}$,使得$m=4k_1+r_1,n=4k_2+r_2$.

所以$a_m=-2+k_1,a_n=-2+k_2$.

所以$a_m+a_n+p=-2+k_1+k_2,a_m+a_n+p+1=-1+k_1+k_2$.

又$m+n=4(k_1+k_2)+r_1+r_2$,所以
当$0\leqslant r_1+r_2<4$ 时, $a_ {m+n}=-2+k_1+k_2$;当$4\leqslant r_1+r_2\leqslant6$时, $a_{m+n}=-1+k_1+k_2$,
所以$a_{m+n}\in\{a_m+a_n+p,a_m+a_n+p+1\}$.

由通项公式可知,当$n\leqslant 9$时, $a_n\leqslant a_{10}=0$;当$n\geqslant 11$时, $a_n\geqslant a_{10}=0$.所以$S_n\geqslant S_{10}$恒成立.

综上,存在$\Re_{p}$数列$\{a_n\}$,使得$S_n\geqslant S_{10}$恒成立,这时$p=2$.
\end{solution}

 


\item (12分) \\
已知函数 $f(x)=\mathrm{e}^x\ln(1+x)$.
\begin{enumerate}[itemsep=-0.3em,label={(\arabic*)},topsep=0pt,labelsep=.5em,leftmargin=3em]
\item 求曲线$y=f(x)$在点$(0,f(0))$处的切线方程;
\item 设$g(x)=f'(x)$,讨论函数$g(x)$在$[0,+\infty)$上的单调性;
\item 证明:对任意的$s,t\in(0,+\infty)$,有$f(s+t)>f(s)+f(t)$.
\end{enumerate}
\begin{solution}
(I)因为$f(x)=\mathrm{e}^x\ln(1+x)$,
所以 $f'(x)=\mathrm{e}^x\left[\frac{1}{1+x}
+\ln(1+x)\right]$. 所以 $f(0)=0,f'(0)=1$.
所以曲线$y=f(x)$在点$(0,f(0))$处的切线方程为$y=x$.

(II)由题设, $g(x)=\mathrm{e} ^x\left[\frac1{1+x}+\ln(1+x)\right]$.
所以
\begin{align*}
g'(x) &=\mathrm{e} ^x\left[\frac{1}{1+x}+\ln(1+x)\right]
+\mathrm{e}^x\left[\frac{1}{1+x}
-\frac{1}{(1+x)^{2}}\right]\\ &=\mathrm{e}^{x}\left[\frac{1+2x}{(1+x)^{2}}
+\ln(1+x)\right].
\end{align*}

因为$x\geqslant 0$,所以$g'(x)>0$.所以函数$g(x)$在$[0,+\infty)$上单调递增.

(III)不妨假设$t>0$取定,令$h(x)=f(x+t)-f(x)-f(t),x\in[0,+\infty)$,
则$h'(x)=f'(x+t)-f'(x),x\in[0,+\infty)$.

由(II)知, $f'(x)$在$[0,+\infty)$上单调递增,
所以$h'(x)=f'(x+t)-f'(x)>0$.

从而$h(x)$在$[0,+\infty)$上单调递增.因为$h(0)=-f(0)=0$,所以当$s>0$时, $h(s)>h(0)=0$,即 $f(s+t)-f(s)-f(t)>0$.

综上,对任意的$s,t\in(0,+\infty)$,有$f(s+t)>f(s)+f(t)$.
\end{solution}

 


\item (12分) \\
已知$Q:a_1,a_2,\cdots,a_k$为有穷整数数列.给定正整数$m$,若对任意的$n\in\{1,2,\cdots,m\}$,在$Q$中存在$a_i,a_{i+1},a_{i+2},\cdots,a_{i+j}\ (j\geqslant 0)$,使得$a_i+a_{i+1}+a_{i+2}+\cdots+a_{i+j}=n$,则称$Q$为$m-$连续可表数列.
\begin{enumerate}[itemsep=-0.3em,label={(\arabic*)},topsep=0pt,labelsep=.5em,leftmargin=3em]
\item 判断$Q:2,1,4$是否为$5-$连续可表数列?是否为$6-$连续可表数列?说明理由;
\item 若$Q:a_1,a_2,\cdots,a_k$为$8-$连续可表数列,求证: $k$的最小值为$4$;
\item 若$Q:a_1,a_2,\cdots,a_k$为$20-$连续可表数列,且$a_1+a_2+\cdots+a_k<20$,求证: $k\geqslant 7$.
\end{enumerate}
\begin{solution}
(I)因为$a_{2}=1,\quad a_{1}=2,\quad a_{1}+a_{2}=3,\quad a_{3}=4,\quad a_{2}+a_{3}=5$,

所以$Q:2,1,4$为$5-$连续可表数列.

又因为$a_1+a_2+a_3=7\neq 6$,所以$Q:2,1,4$不是$6-$连续可表数列.

(II)对于$Q:a_1,a_2,\cdots,a_k$,所有形如$a_i+a_{i+1}+\cdots+a_{i+j}\ \left(i=1,2,\cdots,k;j=0,1,\cdots,k-i\right)$的可能取值最多有 $n(k)=k+(k-1)+\cdots+1=\frac{k(k+1)}2$个.

由题设, $n(k)\geqslant 8$,故$k\geqslant 4$.

对于$Q:1,4,1,2$,因为$a_1=1,\quad a_4=2,\quad a_3+a_4=3,\quad a_2=4,\quad a_1+a_2=5$,
$a_1+a_2+a_3=6,\quad a_2+a_3+a_4=7,\quad a_1+a_2+a_3+a_4=8$,

所以$Q:1,4,1,2$为$8-$连续可表数列.

综上, $k$的最小值为$4$.
\end{solution}

\begin{solution}
(III)由题设, $n(k)\geqslant 20$,
所以$k\geqslant 6$.

假设存在$Q_0:a_1,a_2,\cdots,a_6$为$20-$连续可表数列,且$a_1+a_2+\cdots+a_6<20$.

\ding{172} 如果$Q_{0}$的各项均为非负整数,则$a_i+a_{i+1}+\cdots+a_{i+j}\leqslant a_1+a_2+\cdots+a_6<20$,这与$Q_{_0}$是$20-$连续可表数列矛盾.所以$Q_{0}$有负整数项.

又因为$n(6)=21$,所以$Q_{0}$只有一项为负整数,其余各项均为正整数,且互不相等.

\ding{173} 当$a_1<0$时,形如$a_i+a_{i+1}+\cdots+a_{i+j}$且取值大于$0$的表达式列表如下:
\begin{table}[H]
\centering
\setlength{\tabcolsep}{6mm}{
\begin{tabular}{|l|l|l|l|l|l|}
\hline
$a_2$ & $a_1+a_2$ & & & & \\ \hline
$a_2+a_3$ & $a_1+a_2+a_3$ & $a_3$ & & & \\ \hline
$a_2+a_3+a_4$ & $a_1+a_2+a_3+a_4$ & $a_3+a_4$ & $a_4$ & & \\ \hline
$a_2+a_3+a_4+a_5$ & $a_1+a_2+\cdots +a_5$ & $a_3+a_4+a_5$ & $a_4+a_5$ & $a_5$ & \\ \hline
$a_2+a_3+\cdots+a_6$ & $a_1+a_2+\cdots +a_6$ & $a_3+a_4+a_5+a_6$ & $a_4+a_5+a_6$ & $a_5+a_6$ & $a_6$ \\ \hline
\end{tabular}}
\end{table}

表中表达式的值互不相等,且每一列中的值从上到下增大,每一行中的值从左到右减小,最大值是$a_2+a_3+\cdots+a_6$,且第二大的值是
$$
\max\left\{a_{2}+a_{3}+a_{4}+a_{5},
a_{1}+a_{2}+\cdots+a_{6}\right\}.
$$
由题设,表中所有表达式的值之和为$1+2+\cdots+20=210$,

所以 $5a_1+10(a_2+a_5)+12(a_3+a_4)+6a_6=210$. 故$a_{1}$是偶数,且$a_{1}\leqslant -2$.

由题设, $a_2+a_3+\cdots+a_6=20$ , 所以$a_1+a_2+\cdots+a_6\leqslant 20-2=18$.

所以$a_2+a_3+a_4+a_5=19$.所以$a_6=1$.

因为$a_5>1$,所以$a_2+a_3+a_4<18$,从而$a_1+a_2+\cdots+a_6=18$,

综上得$a_1=-2$, $a_1+a_2+\cdots+a_5=17$.

由題设, $\max\{a_2+a_3+a_4,a_3+a_4+a_5+a_6\}=16$.

又$5a_1+10(a_2+a_5)+12(a_3+a_4)+6a_6=210$,
所以$a_{3}+a_{4}=12,\quad a_{2}+a_{5}=7$.

当$a_{2}+a_{3}+a_{4}=16$时, $a_{2}=4$, $a_{5}=3$.此时$a_{3}+a_{4}+a_{5}+a_{6}=16$,
这与表中表达式的值互不相等矛盾.

当$a_{3}+a_{4}+a_{5}+a_{6}=16$时, $a_{5}=3$, $a_2=4$,此时 $a_{2}+a_{3}+a_{4}=16$,
这与表中表达式的值互不相等矛盾.

所以,当$a_1<0$时, $Q_0$不是$20-$连续可表数列.
\end{solution}

\begin{solution}
\ding{174}当$a_6<0$时,同理可证$Q_{0}$不是$20-$连续可表数列.

\ding{175}当存在$l\in\{2,3,4,5\}$,使得$a_l<0$ 时,由题设, $a_ {l-1}+a_l\geqslant 1,a_l+a_{l+1}\geqslant 1$, 所以$a_i+a_{i+1}+\cdots+a_{i+j}\leqslant a_1+a_2+\cdots+a_6<20$.

所以$Q_{0}$不是$20-$连续可表数列.

综上可知,不存在$6$项的满足题设的$20-$连续可表数列.
所以$k\geqslant 7$.
\end{solution}

\textbf{注:}构造$S=\{1,10,1,2,2,2,2\}$.

$S=\{1,2,4,5,8,-2,-1\}$.

 

构造$S=\{0,1,11,12,14,16,18,20\}$.


(2022年北京高考)已知数列$\{a_n\}$的各项均为正数,其前$n$项和$S_n$满足$a_n\cdot S_n=9\ (n=1,2,\cdots)$.给出下列四个结论:

\ding{172} $\{a_n\}$的第$2$项小于$3$;\quad
\ding{173} $\{a_n\}$为等比数列;

\ding{174} $\{a_n\}$为递减数列;\quad \ding{175} $\{a_n\}$中存在小于$\frac1{100}$的项.

其中所有正确结论的序号是\underline{\hspace{2cm}}.

 


\item (12分) \\
(2023年北京高考)已知椭圆$E:\frac{x^2}{a^2}+\frac{y^2}{b^2}=1\ (a>b>0)$的离心率为$\frac{\sqrt{5}}{3}$, $A$、$C$分别是$E$的上、下顶点, $B$、$D$分别是$E$的左、右顶点, $|AC|=4$.

\begin{enumerate}[itemsep=-0.3em,label={(\arabic*)},topsep=0pt,labelsep=.5em,leftmargin=3em]
\item 求$E$的方程;
\item 设$P$为第一象限内$E$上的动点,直线$PD$与直线$BC$交于点$M$,直线$PA$与直线$y=-2$交于点$N$.求证: $MN\ //\ CD$.
\end{enumerate}
\begin{solution}
1
\end{solution}

北京卷的圆锥曲线大题,前几年的背景都是二次曲线的对合,今年改成了五边形的帕斯卡定理.帕斯卡定理为背景的题目在高考中几乎都以五边形的形式出现,并且计算量相对较大.

已知椭圆$\frac{x^2}{9}+\frac{y^2}{4}=1$的四个顶点分别为 $A,B,C,D$,设$P$是椭圆上一点,直线$l:y=-2$, $PD\cap BC=M$, $PA\cap l=N$.求证: $MN\ //\ CD$.

看到内接五边形并且有切线的,基本上离不开帕斯卡定理了.把五边形视为退化的六边形$CCDPAB$,对它使用帕斯卡定理,得 $CC\cap PA=N$, $CD\cap AB=Q$, $DP\cap BC=M$三点共线.而由于$AB\ //\ CD$, $Q$是无穷远点,因此 $MN\ //\ AB\ //\ CD$.

考试中最直接的想法应该是设$P$的坐标得到直线$AP,DP$后,求出$M,N$坐标,证明$k_{MN}=k_{CD}=\frac{2}{3}$.但这样写会非常复杂,而如果采用同一法来做,计算量将大幅简化.

具体来说,可以直接设$N(t,-2)$并过$N$作$CD$的平行线,交 $BC$于$M'$.如果能证明$M'PD$三点共线,那就说明$M'$就是题目中的$M$,那么$MN\ //\ CD$得证.


设$N(t,-2)$,则$AN:y=-\frac{4}{t}x+2$,联立椭圆得 $(t^2+36)x^2-36tx=0$.

方程两根为$A,P$横坐标,已知$x_A=0$,韦达定理得 $x_P=\frac{36t}{t^2+36}$,因此 $y_P=-\frac{4}{t}\cdot\frac{36t}{t^2+36}+2
=\frac{2t^2-72}{t^2+36}$.

又过$N$作$CD$的平行线交$BC$于$M'$,则 $M'N:3y=2(x-t)-6$.联立 $BC:\frac{x}{-3}+\frac{y}{-2}=1$得 $x_{M'}=\frac{t}{2}$,因此 $y_{M'}=-2-\frac{2}{3}\cdot\frac{t}{2}
=-\frac{6+t}{3}$.

由于 $k_{PD}=\frac{y_P}{x_P-3}=\frac{2t^2-72}{36t-3t^2-108}
=\frac{2(t+6)(t-6)}{-3(t-6)^2}=-\frac{2(t+6)}{3(t-6)}$,
$k_{M'D}=-\frac{\frac{6+t}{3}}{\frac{t}{2}-3}
=-\frac{2(t+6)}{3(t-6)}$,因此 $k_{PD}=k_{M'D}\Rightarrow M'PD$三点共线,得证.

补充说明一下,当$t=6$时上述分母为$0$,这是由于此时$P,D$重合导致的.不过因为原题中限制了$P$在第一象限,所以这种情况可以排除.

 


【命题的背景】(帕斯卡定理)在圆锥曲线$E$上的六点,它们三双对应边的交点共线.

对于本题椭圆$E$上的两组对应点$\left(\frac{ADC}{CBP}\right)$, 三双对应边, $AB$与$CD$交点$H$ ($H$为无穷远点), $AP$与$CC$ (即切线$x=-2$)的交点$N$, $DP$与$BC$的交点$M$, $H$、$M$、$N$三点共线,因$H$是无穷远点, 故$BA$, $CD$, $MN$三线平行.

【双曲线变式题】

设双曲线$E:x^2-y^2=9$的左右顶点分别为$C,A$, 并设 $B(5,4),D(-5,-4)$, $P$是曲线$E$上在第一象限的点, $AP$与$x=-3$交于$N$点, $PD$与$BC$交于$M$,则 $MN\ //\ CD\ //\ AB$.

【抛物线变式题】设抛物线$E:y^{2}=8x$的顶点$C$,并设$B(2,4)$, $D(2,-4)$, $A(8,-8)$, $P$是曲线$E$上在第一象限的点, $PA$与直线$x=0$交于$N$, $PD$与$BC$交于$M$,则$MN\ //\ CD\ //\ AB$.

(2023年北京高考)已知数列$\{a_{n}\}$满足$a_{n+1}=\frac{1}{4}\left(a_{n}-6\right)^{3}+6\ (n=1,2,3,\cdots)$,则

A.当$a_1=3$时, $\{a_n\}$为递减数列,且存在常数$M\leqslant 0$,使得$a_n>M$恒成立

B.当$a_1=5$时, $\{a_n\}$为递增数列,且存在常数$M\leqslant 6$,使得$a_n<M$恒成立

C.当$a_1=7$时, $\{a_n\}$为递减数列,且存在常数$M>6$,使得$a_n>M$恒成立

D.当$a_1=9$时, $\{a_n\}$为递增数列,且存在常数$M>0$,使得$a_n<M$恒成立

 


\item (12分) \\
(2023年北京高考)已知数列$\left\{a_n\right\}$, $\left\{b_n\right\}$的项数均为$m\ \left(m>2\right)$,且$a_n,b_n\in
\left\{1,2,\cdots,m\right\}$, $\left\{a_n\right\}$, $\left\{b_n\right\}$的前$n$项和分别为$A_n,B_n$,并规定$A_0=B_0=0$.对于$k\in\{0,1,2,\cdots,m\}$,定义$r_k=\max\{i|B_i\leqslant A_k,i\in\{0,1,2,\cdots,m\}\}$,其中$\max M$表示数集$M$中最大的数.
\begin{enumerate}[itemsep=-0.3em,label={(\arabic*)},topsep=0pt,labelsep=.5em,leftmargin=3em]
\item 若$a_1=2,a_2=1,a_3=3, b_1=1,b_2=3,b_3=3$,
求$r_0,r_1,r_2,r_3$的值;
\item 若$a_1\geqslant b_1$, 且$2r_j\leqslant r_ {j+1}+r_{j-1},j=1,2,\cdots,m-1$,求$r_n$;
\item 证明:存在$p,q,s,t\in\{0,1,2,\cdots,m\}$,满足$p>q,s>t$,使得$A_p+B_t=A_q+B_s$.
\end{enumerate}
\begin{solution}
1
\end{solution}

(2023年天津高考)设$a\in\mathbf{R}$,函数$f(x)=ax^2-2x-|x^2-ax+1|$.若$f(x)$恰有两个零点,则$a$的取值范围为\underline{\hspace{2cm}}.

\textbf{解:}令 $g(x)=ax^2-2x$, $h(x)=x^2-ax+1$, $h(x)$的判别式$\Delta_1=a^2-4$.

(i)当$\Delta_1\leqslant 0$,即$|a|\leqslant 2$时, $f(x)=ax^2-2x-(x^2-ax+1)=(a-1)x^2+(a-2)+1$.

\ding{172} 当$a=1$时, $f(x)=-x-1$恰有一个零点,不合题意.

\ding{173} 当$a\neq1$时, $f(x)$的判别式 $\Delta_2=(a-2)^2+4(a-1)=a^2\geqslant 0$.当 $\Delta_2=0$,即 $a=0$时, $f(x)=-(x+1)^2$恰有一个零点,不合题意;当$\Delta_2>0$,即$a\neq0$时, $f(x)$恰有两个零点.

(ii)当$\Delta_1>0$,即$|a|>2$时,若$f(x)=0$,则有$ax^2-2x=|x^2-ax+1|$,故$(ax^2-2x)^2=(x^2-ax+1)^2$,整理得
$$
(x-1)(x+1)((a-1)x-1)((a+1)x-1)=0.
$$
从而$f(x)=0\Leftrightarrow x\in\left\{1,-1,\frac{1}{a-1},\frac{1}{a+1}\right\}$ 且$g(x)\geqslant 0$.注意到
$$\begin{cases}
g(1)=a-2,\\
g(-1)=a+2,\\
g\left(\frac{1}{a-1}\right)=-\frac{a-2}{(a-1)^2},\\
g\left(\frac{1}{a+1}\right)=-\frac{a+2}{(a+1)^2},
\end{cases}$$
所以,当$|a|>2$时, $f(x)$恰有两个零点.

由(i) (ii)可知, $f(x)$恰有两个零点当且仅当 $a\in(-\infty,0)\cup(0,1)\cup(1,+\infty)$.

 

\item (12分) \\
(2023年天津高考)已知函数 $f(x)=\left(\frac{1}{x}+\frac{1}{2}\right)\ln(x+1)$.
\begin{enumerate}[itemsep=-0.3em,label={(\arabic*)},topsep=0pt,labelsep=.5em,leftmargin=3em]
\item 求曲线$y=f(x)$在$x=2$处的切线斜率;
\item 求证:当$x>0$时, $f(x)>1$;
\item 求证: $\frac{5} {6}<\ln(n!)-\left(n+\frac{1}{2}\right)\ln n+n\leqslant 1\ (n\in\mathbf{N}^\ast)$.
\end{enumerate}
\begin{solution}
(I) \textbf{解:} 由已知,可得$f'(x)=-\frac{1}{x}\ln(x+1)+\frac{\frac{1}{x}+\frac{1}{2}}{x+1}$,故有$f'(2)=-\frac{\ln3}{4}+\frac{1}{3}$.所以,曲线$y=f(x)$在$x=2$处的切线斜率为$\frac{4-3\ln3}{12}$.

(II) \textbf{证明:}由于$x>0$,故$f(x)>1$等价于$\ln(x+1)>\frac{2x}{x+2}$.令
$$
g(x)=\ln(x+1)-\frac{2x}{x+2},\quad x\in [0,+\infty),
$$
可得$g'(x)=\frac{1}{x+1}-\frac{2}{x+2}+\frac{2x}{(x+2)^{2}}
=\frac{x^{2}}{(x+1)(x+2)^{2}}$,
因此当$x>0$时, $g'(x)>0$,由此可得$g(x)$在 $[0,+\infty)$单调递增.所以,当$x>0$时, $g(x)>g(0)=0$,即$f(x)>1$.
\end{solution}

\begin{solution}
(III) \textbf{证明:}当$n=1$时, $\ln(n!)-\left(n+\frac12\right)\ln n+n=1$,结论成立.

当$n\geqslant 2$时,由$f\left(\frac{1}{k}\right)=\left(k+\frac{1}{2}\right)(\ln(k+1)-\ln k)$ (其中$k\in\mathbf{N}^\ast)$,有
\begin{align*}
\sum_{k=1}^{n-1}f\left(\frac{1}{k}\right)& =\sum_{k=1}^{n-1}\left(\left(k+\frac{3}{2}\right)
\ln(k+1)-\left(k+\frac{1}{2}\right)\ln k-\ln(k+1)\right) \\
&=\left(n+\frac12\right)\ln n-\sum_{k=1}^{n-1}\ln(k+1) \\
&=-\ln(n!)+\left(n+\frac12\right)\ln n.
\end{align*}

一方面,由$f\left(\frac1k\right)>1$,可得$\sum_{k=1}^{n-1}f\left(\frac{1}{k}\right)>n-1$,
即$-\ln(n!)+\left(n+\frac{1}{2}\right)\ln n>n-1$,亦即$\ln(n!)-\left(n+\frac{1}{2}\right)\ln n+n<1$.

另一方面,令$h(x)=\ln(x+1)-\frac{2x}{x+2}\left(1+\frac{x^2}{12}\right)$, $x\in[0,+\infty)$,

则当$x>0$时, $h'(x)=-\frac{x^3(x+4)}{3(x+1)(x+2)^2}<0$,故 $h(x)$在$[0,+\infty)$单调递减.

所以当$x>0$时, $h(x)<h(0)=0$,进而可得 $f(x)<1+\frac{x^{2}}{12}$.


因此,当$n\geqslant 2$时有$\sum_{k=1}^{n-1}f\left(\frac1k\right)
<n-1+\frac1{12}\sum_{k=1}^{n-1}\frac1{k^2}$,
其中, $\sum_ {k=1}^{n-1}\frac1{k^2}
=1+\sum_{k=2}^{n-1}\frac1{k^2}
<1+\sum_{k=2}^{n-1}\frac1{k(k-1)}
=1+\sum_{k=2}^{n-1}\left(\frac1{k-1}-\frac1k\right)
=2-\frac1{n-1}<2$.

所以$\sum_{k=1}^{n-1}f\left(\frac1k\right)<n-1+\frac1{12}
\sum_{k=1}^{n-1}\frac1{k^2}<n-\frac56$, 即$\ln(n!)-\left(n+\frac12\right)\ln n+n>\frac56$.

故对于任意的正整数$n$,有$\ln(n!)-\left(n+\frac12\right)\ln n+n>\frac56$.

综上,对于任意的正整数$n$,都有 $\frac56<\ln(n!)-\left(n+\frac12\right)\ln n+n\leqslant 1$.
\end{solution}

 

 

\item (12分) \\
(2023年天津高考)已知$\{a_n\}$是等差数列, $a_2+a_5=16$, $a_5-a_3=4$.
\begin{enumerate}[itemsep=-0.3em,label={(\arabic*)},topsep=0pt,labelsep=.5em,leftmargin=3em]
\item 求$\{a_n\}$的通项公式及$\sum_{i=2^{n-1}}^{2^n-1}a_i\ (n\in\mathbf{N}^\ast)$;
\item 设 $\{b_n\}$是等比数列,且对任意的 $k\in\mathbb{N}^\ast$,当 $2^{k-1}\leqslant n\leqslant 2^k-1$时, $b_k<a_n<b_{k+1}$.

(i)当$k\geqslant 2$时,求证: $2^k-1<b_k<2^k+1$;

(ii)求$\{b_n\}$的通项公式及前$n$项和.
\end{enumerate}
\begin{solution}
(I) \textbf{解:}设$\{a_n\}$的公差为$d$,由$a_2+a_5=16$可得 $2a_1+5d=16$, 由$a_5-a_3=4$,可得$2d=4$,故可解得 $a_1=3$, $d=2$, 由此可得 $a_n=2n+1$.

因此有 $a_{2^{n-1}}=2^n+1, a_{2^n-1}=2^{n+1}-1$, 故
$$
\sum_{i=2^{n-1}}^{2^n-1}a_i
=\frac{(2^n+1+2^{n+1}-1)\times
2^{n-1}}2=3\times 2^{2n-2}.
$$

(II)\textbf{解:} (i)由已知,当$2^{k-1}\leqslant n\leqslant 2^k-1$时,有$b_k<a_n$且$b_{k+1}>a_n$.

由此可得$b_k<a_{2^{k-1}}=2^k+1$,且$b_{k+1}>a_{2^k-1}=2^{k+1}-1$.

所以,对于任意的$k\geqslant 2$, $2^k-1<b_k<2^k+1$.

(ii)设$\{b_n\}$的公比为$q$.由(i),对于任意的$k\geqslant 2$有$2^{k+1}-1<b_{k+1}<2^{k+1}+1$.

由此可得$\frac{2^{k+1}-1}{2^k+1}<\frac{b_{k+1}}{b_k}=q
<\frac{2^{k+1}+1}{2^k-1}$,
即$2-\frac{3}{2^k+1}<q<2+\frac{3}{2^k-1}$,
进而对于任意的$k\geqslant 2$,有$|q-2|<\frac4{2^k}$.

假如$q\neq2$,则当$k>\log_2\frac4{|q-2|}$时,有$|q-2|>\frac4{2^k}$,矛盾,所以, $q=2$.

由$2^k-1<b_k=b_1\times 2^{k-1}<2^k+1$,
有$2-\frac1{2^{k-1}}<b_1<2+\frac1{2^{k-1}}$,
故$|b_1-2|<\frac1{2^{k-1}}<\frac4{2^k}$.

同上可得$b_{1}=2$.

所以, $b_n=2^n$.因此, $\{b_n\}$的前$n$项和为 $S_n=\frac{2(1-2^n)}{1-2}=2^{n+1}-2$.
\end{solution}

$\frac{\pi}{4}-\sum_{k=1}^{n}\frac{n}{n^2+k^2}$


\item (12分) \\
(2024年合肥二模)在数学中,广义距离是泛函分析中最基本的概念之一.对平面直角坐标系中两个点$P_{1}\left(x_{1},y_{1}\right)$
和$P_{2}\left(x_{2},y_{2}\right)$,记
$$\left|P_{1}P_{2}\right|_{t}=\max\left\{\frac{\left|x_{1}-x_{2}\right|}{1+\left|x_{1}-x_{2}\right|},
\frac{\left|y_{1}-y_{2}\right|}{1+\left|y_{1}-y_{2}\right|}\right\},$$
称$|P_1P_2|_t$为点$P_1$与点$P_{2}$之间的“$t-$距离”,其中$\max\{p,q\}$表示$p,q$中较大者.
\begin{enumerate}[itemsep=-0.3em,label={(\arabic*)},topsep=0pt,labelsep=.5em,leftmargin=3em]
\item[(1)] 计算点$P(1,2)$和点$Q(2,4)$之间的“$t-$距离”;
\item[(2)] 设$P_0(x_0,y_0)$是平面中一定点, $r>0$.我们把平面上到点$P_{0}$的“$t-$距离”为$r$的所有点构成的集合叫做以点$P_{0}$为圆心,以$r$为半径的“$t-$圆”.求以原点$O$ 为圆心,以$\frac{1}{2}$为半径的“$t-$圆”的面积;
\item[(3)] 证明:对任意点$P_1(x_1,y_1),P_2(x_2,y_2),P_3(x_3,y_3)$, $|P_1P_3| _t\leqslant\left|P_1P_2\right|_t+\left|P_2P_3\right|_t$.
\end{enumerate}
\begin{solution}
1
\end{solution}

 

 

\item (12分) \\
已知函数 $f(x)=ax^2-x+\ln(x+1)$, $a\in\mathbf{R}$.
\begin{enumerate}[itemsep=-0.3em,label={(\arabic*)},topsep=0pt,labelsep=.5em,leftmargin=3em]
\item[(1)] 若对定义域内任意非零实数$x_1,x_2$,均有$\frac{f(x_1)f(x_2)}{x_1x_2}>0$,求$a$;
\item[(2)] 记$t_n=1+\frac12+\cdots+\frac1n$,证明: $t_n-\frac56<\ln(n+1)<t_n$.
\end{enumerate}
\begin{solution}
(1)解: $f(x)$的定义域为$\left(-1,+\infty\right)$,且$f(0)=0$;

$f'(x)=2ax-1+\frac{1}{x+1}=2ax-\frac{x}{x+1}
=x(2a-\frac{1}{x+1})$,因此$f'(0)=0$;

(i)当$a\leqslant 0$时, $2a-\frac1{x+1}<0$,则此时令 $f'(x)>0$有 $x\in(-1,0)$,令 $f'(x)<0$有 $x\in(0,+\infty)$,则$f(x)$在$(-1,0)$上单调递增, $(0,+\infty)$上单调递减,又$f(0)=0$,于是$f(x)\leqslant 0$,此时令$x_1x_2<0$,有$\frac{f(x_1)f(x_2)}{x_2x_2}<0$,不符合题意;

(ii)当$a>0$时, $f'(x)$有零点$0$和$x_{0}=\frac{1}{2a}-1$,

若$x_0<0$,即$a>\frac12$,此时令$f'(x)<0$有$x\in(x_0,0)$, $f(x)$在$(x_0,0)$上单调递减,又$f(0)=0$,则$f(x_0)>0$,令$x_1>0$, $x_2=x_0$,有$\frac{f(x_1)f(x_2)}{x_1x_2}<0$,不符合题意;

若$x_0>0$,即$0<a<\frac12$,此时令$f'(x)<0$有$x\in(0,x_0)$, $f(x)$在$(0,x_0)$上单调递减,又 $f(0)=0$,则 $f(x_0)<0$ , 令$-1<x_1<0$, $x_2=x_0$,有$\frac{f(x_1)f(x_2)}{x_1x_2}<0$, 不符合题意;

若$x_0=0$,即$a=\frac{1}{2}$,此时$f'(x)=\frac{x^2}{x+1}>0$, $f(x)$在$(-1,+\infty)$上单调递增,又$f(0)=0$,则$x>0$时$f(x)>0$, $x<0$ 时$f(x)<0$;则$x\neq 0$时$\frac{f(x)}x>0$,也即对$x_1x_2\neq0$, $\frac{f(x_1)f(x_2)}{x_1x_2}>0$,

综上, $a=\frac{1} {2}$.

(2)证:由(1)问的结论可知, $a=0$时, $f(x)=-x+\ln(x+1)\leqslant 0$ ;

$a=\frac{1}{2}$且$x>0$时, $f(x)=\frac{1} {2}x^{2}-x+\ln(x+1)>0$,

则$x>0$时, $x-\frac{1} {2}x^{2}<\ln(x+1)<x$,令$x=\frac{1}{n}$,
有$\frac1n-\frac1{2n^2}<\ln\left(\frac1n+1\right)<\frac1n$, 即$\frac1n-\frac1{2n^2}<\ln(n+1)-\ln n<\frac1n$,

于是$\frac{1}{n-1}-\frac{1}{2(n-1)^{2}}<\ln n-\ln(n-1)<\frac{1}{n-1}$, $\cdots$, $1-\frac{1}{2}<\ln2<1$.

将上述$n$个式子相加,得$t_n-\frac12\left(1+\frac1{2^2}+\cdots+\frac1{n^2}\right)
<\ln(n+1)<t_n$.

欲证$t_n-\frac56<\ln(n+1)<t_n$,只需证$t_n-\frac56<t_n-\frac12(1+\frac1{2^2}+\cdots
+\frac1{n^2})$,只需证$1+\frac1{2^2}+\cdots+\frac1{n^2}<\frac53$.

因为$\frac1{n^2}=\frac4{4n^2}<\frac4{4n^2-1}=2\left(\frac1{2n-1}-\frac1{2n+1}\right)$,

所以$1+\frac{1}{2^{2}}+\cdots+\frac{1}{n^{2}}
<1+2\left(\frac{1}{3}-\frac{1}{5}+\frac{1}{5}
-\frac{1}{7}+\cdots+\frac{1}{2n-1}-\frac{1}{2n+1}\right)
=\frac{5}{3}-\frac{2}{2n+1}<\frac{5}{3}$, 得证.

于是$t_n-\frac56<\ln(n+1)<t_n$得证.
\end{solution}

 

 

\item (12分) \\
已知函数 $f(x)=\sin x-\ln(1+x)$, $f'(x)$为 $f(x)$ 的导数.证明:
\begin{enumerate}[itemsep=-0.3em,label={(\arabic*)},topsep=0pt,labelsep=.5em,leftmargin=3em]
\item[(1)] $f'(x)$在区间$\left(-1,\frac\pi2\right)$存在唯一极大值点;
\item[(2)] $f(x)$有且仅有$2$个零点.
\end{enumerate}
\begin{solution}
(1)设$g(x)=f'(x)$,则$g(x)=\cos x-\frac{1}{1+x}$, $g'(x)=-\sin x+\frac{1}{\left(1+x\right)^{2}}$.

当$x\in(-1,\frac\pi2)$时, $g'(x)$单调递减,而$g'(0)>0$, $g'\left(\frac\pi2\right)<0$, 可得$g'(x)$在$\left(-1,\frac\pi2\right)$有唯一零点,设为$\alpha$.则当$x\in(-1,\alpha)$时, $g'(x)>0$;当$x\in \left(\alpha,\frac\pi2\right)$时, $g'(x)<0$.

所以$g(x)$在$(-1,\alpha)$单调递增,在$\left(\alpha,\frac\pi2\right)$单调递减,故$g(x)$在$\left(-1,\frac\pi2\right)$存在唯一极大值点,即$f'(x)$在$\left(-1,\frac\pi2\right)$存在唯一极大值点.


(2) $f(x)$的定义域为$(-1,+\infty$).

(i)当$x\in(-1,0]$时,由(1)知, $f'(x)$在$(-1,0)$单调递增,而$f'(0)=0$,所以当$x\in(-1,0)$时, $f'(x)<0$,故$f(x)$在$(-1,0)$单调递减.又$f(0)=0$,从而$x=0$是$f(x)$在$(-1,0]$的唯一零点.

(ii)当$x\in\left(0,\frac\pi2\right]$时,
由(1)知, $f'(x)$在$(0,\alpha)$单调递增,在$\left(\alpha,\frac\pi2\right)$单调递减,
而$f'(0)=0$, $f'\left(\frac\pi2\right)<0$,所以存在$\beta\in\left(\alpha,\frac\pi2\right)$,
使得$f'(\beta)=0$,且当$x\in(0,\beta)$时, $f'(x)>0$;当$x\in\left(\beta,\frac\pi2\right)$时, $f'(x)<0$.故$f(x)$在$(0,\beta)$单调递增,在$\left(\beta,\frac\pi2\right)$单调递减.

又$f(0)=0$, $f\left(\frac\pi2\right)
=1-\ln\left(1+\frac\pi2\right)>0$,所以当$x\in\left(0,\frac\pi2\right]$时, $f(x)>0$.从而, $f(x)$在$\left(0,\frac\pi2\right]$没有零点.

(iii)当$x\in \left(\frac\pi2,\pi\right]$时, $f'(x)<0$,所以$f(x)$在$\left(\frac\pi2,\pi\right)$单调递减.而$f\left(\frac\pi2\right)>0$, $f(\pi)<0$,所以$f(x)$在$\left(\frac\pi2,\pi\right]$有唯一零点.

(iv) 当$x\in(\pi,+\infty)$时, $\ln(x+1)>1$,所以$f(x)<0$,从而$f(x)$在$(\pi,+\infty)$没有零点.

综上, $f(x)$有且仅有$2$个零点.
\end{solution}

 

\item (12分) \\
为治疗某种疾病,研制了甲、乙两种新药,希望知道哪种新药更有效,为此进行动物试验.试验方案如下:每一轮选取两只白鼠对药效进行对比试验.对于两只白鼠,随机选一只施以甲药,另一只施以乙药.一轮的治疗结果得出后,再安排下一轮试验.当其中一种药治愈的白鼠比另一种药治愈的白鼠多4只时,就停止试验,并认为治愈只数多的药更有效.为了方便描述问题,约定:对于每轮试验,若施以甲药的白鼠治愈且施以乙药的白鼠未治愈则甲药得1分,乙药得$-1$分;若施以乙药的白鼠治愈且施以甲药的白鼠未治愈则乙药得1分,甲药得$-1$分;若都治愈或都未治愈则两种药均得0分.甲、 乙两种药的治愈率分别记为$\alpha$和$\beta$,一轮试验中甲药的得分记为$X$.
\begin{enumerate}[itemsep=-0.3em,label={(\arabic*)},topsep=0pt,labelsep=.5em,leftmargin=3em]
\item[(1)] 求$X$的分布列;
\item[(2)] 若甲药、乙药在试验开始时都赋予4 分, $p_i\ (i=0,1,\cdots,8)$表示“甲药的累计得分为$i$时,最终认为甲药比乙药更有效”的概率,则$p_0=0$, $p_8=1,p_i=ap_{i-1}+bp_i+cp_{i+1}\ (i=1,2,\cdots,7)$,其中$a=P(X=-1),b=P(X=0),c=P(X=1)$.假设$\alpha=0.5$, $\beta=0.8$.

(i)证明: $\{p_ {i+1}-p_i\}\ \left(i=0,1,2,\cdots,7\right)$为等比数列;

(ii)求$p_4$,并根据$p_{4}$的值解释这种试验方案的合理性.
\end{enumerate}
\begin{solution}
(1) $X$的所有可能取值为$-1,0,1$.

$P(X=-1)=(1-\alpha)\beta$, $P(X=0)=\alpha\beta+(1-\alpha)(1-\beta)$,
$P(X=1)=\alpha\left(1-\beta\right)$.

所以$X$ 的分布列为
\begin{table}[H]
\centering
\setlength{\tabcolsep}{6mm}{
\begin{tabular}{|c|c|c|c|}
\hline
$X$ & $-1$ & $0$ & $1$\\ \hline
$P$ & $(1-\alpha)\beta$ & $\alpha\beta+(1-\alpha)(1-\beta)$ & $\alpha\left(1-\beta\right)$ \\ \hline
\end{tabular}}
\end{table}


(2) (i)由(1)得$a=0.4,b=0.5,c=0.1$.

因此$p_i=0.4p_{i-1}+0.5p_i+0.1p_{i+1}$,
故$0.1(p_{i+1}-p_i)=0.4(p_i-p_{i-1})$,即
$$
p_{i+1}-p_{i}=4(p_{i}-p_{i-1}).
$$

又因为$p_1-p_0=p_1\neq 0$,所以$\{p_{i+1}-p_i\}\ (i=0,1,2,\cdots,7)$为公比为$4$,首项为$p_1$的等比数列.

(ii)由(i)可得
\begin{align*}
p_{8} &=p_{8}-p_{7}+p_{7}-p_{6}+\cdots+p_{1}-p_{0}+p_{0}\\ &=(p_{8}-p_{7})+(p_{7}-p_{6})+\cdots+(p_{1}-p_{0})\\ &=\frac{4^8-1}{3}p_1.
\end{align*}

由于$p_8=1$,故$p_1=\frac 3{4^{8}- 1}$,所以
\begin{align*}
p_{4}&=(p_{4}-p_{3})+(p_{3}-p_{2})+(p_{2}-p_{1})
+(p_{1}-p_{0})\\
&=\frac{4^4-1}{3}p_{1}={\frac{1}{257}}.
\end{align*}

$p_4$表示最终认为甲药更有效的概率.由计算结果可以看出,在甲药治愈率为0.5, 乙药治愈率为0.8时,认为甲药更有效的概率为$p_{4}=\frac{1}{257}\approx0.0039$,此时得出错误结论的概率非常小,说明这种试验方案合理.
\end{solution}


\item (12分) \\
(2021年新高考2卷)一种微生物群体可以经过自身繁殖不断生存下来,设一个这种微生物为第0代,经过一次繁殖后为第1代,再经过一次繁殖后为第2代, $\cdots$,该微生物每代繁殖的个数是相互独立的且有相同的分布列,设$X$表示1个微生物个体繁殖下一代的个数, $P(X=i)=p_{i}\ (i=0,1,2,3)$.
\begin{enumerate}[itemsep=-0.3em,label={(\arabic*)},topsep=0pt,labelsep=.5em,leftmargin=3em]
\item[(1)] 已知$p_0=0.4,p_1=0.3,p_2=0.2,p_3=0.1$,求$E(X)$;
\item[(2)] 设$p$表示该种微生物经过多代繁殖后临近灭绝的概率, $p$是关于$x$的方程: $p_0+p_1x+p_2x^2+p_3x^3=x$的一个最小正实根,求证: 当$E(X)\leqslant 1$时, $p=1$,当$E(X)>1$时, $p<1$;
\item[(3)] 根据你的理解说明(2)问结论的实际含义.
\end{enumerate}
\begin{solution}
1
\end{solution}

\end{enumerate}


%%%%%%%%%%%%%%%%%%%%%%%%%%%%%%%%%%%%%%%%%%%%%%%%%%%%%%%%%%%%%%%%%%%%%%%%%%%%%%%
%------------------------------------结束--------------------------------------
%%%%%%%%%%%%%%%%%%%%%%%%%%%%%%%%%%%%%%%%%%%%%%%%%%%%%%%%%%%%%%%%%%%%%%%%%%%%%%%
\clearpage

\end{document}

posted on 2024-09-02 08:38  Eufisky  阅读(18)  评论(0编辑  收藏  举报

导航